LSAT and Law School Admissions Forum

Get expert LSAT preparation and law school admissions advice from PowerScore Test Preparation.

 Administrator
PowerScore Staff
  • PowerScore Staff
  • Posts: 8917
  • Joined: Feb 02, 2011
|
#40669
Complete Question Explanation
(The complete setup for this game can be found here: lsat/viewtopic.php?t=15480)

The correct answer choice is (A)

This question is relatively easy if you identified that K can only be shown third or fourth. Because J must be shown sixth or seventh, J and K can never be consecutive, and answer choice (A) cannot be true and is therefore correct.

Note that answer choice (B) is proven possible by the hypothetical produced in question #1.

Get the most out of your LSAT Prep Plus subscription.

Analyze and track your performance with our Testing and Analytics Package.